how to prove supremum and infimum












0












$begingroup$


I started to learn this subject, and I have understood what It's mean, but I cant find out how to find and to prove it.
for example :



let $X$ be the set of all rational number in form $m/n$ so $ 0<m<n $
prove that $inf X = 0$ and $sup X =1$, and prove that $max X$ and $min X$ does not exist.



how can I solve this?
can you please explain it step by step so I would understand










share|cite|improve this question











$endgroup$












  • $begingroup$
    $sup X = 0$? 1/2 sits in $X$.
    $endgroup$
    – Gibbs
    Nov 26 '17 at 16:48
















0












$begingroup$


I started to learn this subject, and I have understood what It's mean, but I cant find out how to find and to prove it.
for example :



let $X$ be the set of all rational number in form $m/n$ so $ 0<m<n $
prove that $inf X = 0$ and $sup X =1$, and prove that $max X$ and $min X$ does not exist.



how can I solve this?
can you please explain it step by step so I would understand










share|cite|improve this question











$endgroup$












  • $begingroup$
    $sup X = 0$? 1/2 sits in $X$.
    $endgroup$
    – Gibbs
    Nov 26 '17 at 16:48














0












0








0





$begingroup$


I started to learn this subject, and I have understood what It's mean, but I cant find out how to find and to prove it.
for example :



let $X$ be the set of all rational number in form $m/n$ so $ 0<m<n $
prove that $inf X = 0$ and $sup X =1$, and prove that $max X$ and $min X$ does not exist.



how can I solve this?
can you please explain it step by step so I would understand










share|cite|improve this question











$endgroup$




I started to learn this subject, and I have understood what It's mean, but I cant find out how to find and to prove it.
for example :



let $X$ be the set of all rational number in form $m/n$ so $ 0<m<n $
prove that $inf X = 0$ and $sup X =1$, and prove that $max X$ and $min X$ does not exist.



how can I solve this?
can you please explain it step by step so I would understand







supremum-and-infimum






share|cite|improve this question















share|cite|improve this question













share|cite|improve this question




share|cite|improve this question








edited Nov 26 '17 at 16:53









AJY

4,14521128




4,14521128










asked Nov 26 '17 at 16:46









Moran TailuMoran Tailu

92




92












  • $begingroup$
    $sup X = 0$? 1/2 sits in $X$.
    $endgroup$
    – Gibbs
    Nov 26 '17 at 16:48


















  • $begingroup$
    $sup X = 0$? 1/2 sits in $X$.
    $endgroup$
    – Gibbs
    Nov 26 '17 at 16:48
















$begingroup$
$sup X = 0$? 1/2 sits in $X$.
$endgroup$
– Gibbs
Nov 26 '17 at 16:48




$begingroup$
$sup X = 0$? 1/2 sits in $X$.
$endgroup$
– Gibbs
Nov 26 '17 at 16:48










3 Answers
3






active

oldest

votes


















0












$begingroup$

Here the supremum case: For any positive integers $m,n$ with $m<n$, then $m/n<1$, so we get immediately $sup Xleq 1$. Now we claim that $sup X=1$. Given $epsilon>0$, by Archimedean property, we can find some positive integer $N$ such that $1/N<epsilon$, then $(N-1)/Nin X$ and satisfies $(N-1)/N=1-1/N>1-epsilon$, so $sup X>1-epsilon$. Since this is true for all $epsilon>0$, then $sup Xgeq 1$, so we get $sup X=1$.



Similar argument using Archimedian property will show that $inf X=0$.






share|cite|improve this answer









$endgroup$





















    0












    $begingroup$

    So your set is
    $$X = bigg{frac{n}{m}: n, m in mathbb{N} mbox{ and } 0 < m < nbigg}.$$
    Claim: $inf X = 0$ and $sup X = 1$. You can easily see that both of them do not sit in $X$ as $m$ cannot be $0$ and $m$ cannot be equal to $n$. So in case we prove our claim, we can say that $0$ and $1$ are not minimum and maximum of $X$. This is because maxima and minima are required to be in your set in general.
    To check that $0$ is the infimum you can check two properties: that $0$ is smaller than every element of your set, and that if you pick $varepsilon > 0$ you can always find an element of your set between $0$ and $0+varepsilon$.



    Clearly $0 < frac{m}{n}$ as $m,n > 0$. So let us find $frac{m}{n} < varepsilon$. Let us fix $l in mathbb{N}, l > 1$ large enough so that $lvarepsilon > 1$. Such an $l$ exists by the Archimedean property of the real numbers. So $frac{1}{l} < varepsilon$. This proves your first claim, $inf X = 0$.



    You can do a similar proof for $sup X = 1$. It is enough to prove that $1$ is greater than all the elements in $X$, and that once you fix $varepsilon > 0$ you can always find an element of $X$ between $1-varepsilon$ and $1$.






    share|cite|improve this answer









    $endgroup$





















      0












      $begingroup$

      First, we show the infimum and supernumerary are as you say. Clearly $0$ is a lower bound, and $1$ an upper bound, but we must show they are, respectively, the greatest lower bound and least upper bound. I claim that (i) if $a > 0$ is a rational number, then there exists $x in X$ such that $x < a$, and (ii) if $b < 1$ is a rational number, then there exists $y in X$ such that $y > b$.



      For (i), let $a = m / n$, where $0 < m < n$. We wanna find a number between $0$ and $a$, so we can make the easy choice and pick the midpoint between them, namely $x = frac{m}{2n}$. Thus if $a$ was



      For (ii), let $b = p / q, 0 < p < q$, and then let $ y = frac{2p + 1}{2 q}$. Again, we choose a point between $b$ and $1$ to show $b$ wasn't an upper bound.



      So now we've determined that $inf X = 0, sup X = 1$. But neither $0$ nor $1$ is contained in $X$, so they can't be the minimum or maximum, respectively.






      share|cite|improve this answer









      $endgroup$













        Your Answer





        StackExchange.ifUsing("editor", function () {
        return StackExchange.using("mathjaxEditing", function () {
        StackExchange.MarkdownEditor.creationCallbacks.add(function (editor, postfix) {
        StackExchange.mathjaxEditing.prepareWmdForMathJax(editor, postfix, [["$", "$"], ["\\(","\\)"]]);
        });
        });
        }, "mathjax-editing");

        StackExchange.ready(function() {
        var channelOptions = {
        tags: "".split(" "),
        id: "69"
        };
        initTagRenderer("".split(" "), "".split(" "), channelOptions);

        StackExchange.using("externalEditor", function() {
        // Have to fire editor after snippets, if snippets enabled
        if (StackExchange.settings.snippets.snippetsEnabled) {
        StackExchange.using("snippets", function() {
        createEditor();
        });
        }
        else {
        createEditor();
        }
        });

        function createEditor() {
        StackExchange.prepareEditor({
        heartbeatType: 'answer',
        autoActivateHeartbeat: false,
        convertImagesToLinks: true,
        noModals: true,
        showLowRepImageUploadWarning: true,
        reputationToPostImages: 10,
        bindNavPrevention: true,
        postfix: "",
        imageUploader: {
        brandingHtml: "Powered by u003ca class="icon-imgur-white" href="https://imgur.com/"u003eu003c/au003e",
        contentPolicyHtml: "User contributions licensed under u003ca href="https://creativecommons.org/licenses/by-sa/3.0/"u003ecc by-sa 3.0 with attribution requiredu003c/au003e u003ca href="https://stackoverflow.com/legal/content-policy"u003e(content policy)u003c/au003e",
        allowUrls: true
        },
        noCode: true, onDemand: true,
        discardSelector: ".discard-answer"
        ,immediatelyShowMarkdownHelp:true
        });


        }
        });














        draft saved

        draft discarded


















        StackExchange.ready(
        function () {
        StackExchange.openid.initPostLogin('.new-post-login', 'https%3a%2f%2fmath.stackexchange.com%2fquestions%2f2538207%2fhow-to-prove-supremum-and-infimum%23new-answer', 'question_page');
        }
        );

        Post as a guest















        Required, but never shown

























        3 Answers
        3






        active

        oldest

        votes








        3 Answers
        3






        active

        oldest

        votes









        active

        oldest

        votes






        active

        oldest

        votes









        0












        $begingroup$

        Here the supremum case: For any positive integers $m,n$ with $m<n$, then $m/n<1$, so we get immediately $sup Xleq 1$. Now we claim that $sup X=1$. Given $epsilon>0$, by Archimedean property, we can find some positive integer $N$ such that $1/N<epsilon$, then $(N-1)/Nin X$ and satisfies $(N-1)/N=1-1/N>1-epsilon$, so $sup X>1-epsilon$. Since this is true for all $epsilon>0$, then $sup Xgeq 1$, so we get $sup X=1$.



        Similar argument using Archimedian property will show that $inf X=0$.






        share|cite|improve this answer









        $endgroup$


















          0












          $begingroup$

          Here the supremum case: For any positive integers $m,n$ with $m<n$, then $m/n<1$, so we get immediately $sup Xleq 1$. Now we claim that $sup X=1$. Given $epsilon>0$, by Archimedean property, we can find some positive integer $N$ such that $1/N<epsilon$, then $(N-1)/Nin X$ and satisfies $(N-1)/N=1-1/N>1-epsilon$, so $sup X>1-epsilon$. Since this is true for all $epsilon>0$, then $sup Xgeq 1$, so we get $sup X=1$.



          Similar argument using Archimedian property will show that $inf X=0$.






          share|cite|improve this answer









          $endgroup$
















            0












            0








            0





            $begingroup$

            Here the supremum case: For any positive integers $m,n$ with $m<n$, then $m/n<1$, so we get immediately $sup Xleq 1$. Now we claim that $sup X=1$. Given $epsilon>0$, by Archimedean property, we can find some positive integer $N$ such that $1/N<epsilon$, then $(N-1)/Nin X$ and satisfies $(N-1)/N=1-1/N>1-epsilon$, so $sup X>1-epsilon$. Since this is true for all $epsilon>0$, then $sup Xgeq 1$, so we get $sup X=1$.



            Similar argument using Archimedian property will show that $inf X=0$.






            share|cite|improve this answer









            $endgroup$



            Here the supremum case: For any positive integers $m,n$ with $m<n$, then $m/n<1$, so we get immediately $sup Xleq 1$. Now we claim that $sup X=1$. Given $epsilon>0$, by Archimedean property, we can find some positive integer $N$ such that $1/N<epsilon$, then $(N-1)/Nin X$ and satisfies $(N-1)/N=1-1/N>1-epsilon$, so $sup X>1-epsilon$. Since this is true for all $epsilon>0$, then $sup Xgeq 1$, so we get $sup X=1$.



            Similar argument using Archimedian property will show that $inf X=0$.







            share|cite|improve this answer












            share|cite|improve this answer



            share|cite|improve this answer










            answered Nov 26 '17 at 17:05









            user284331user284331

            35.3k31646




            35.3k31646























                0












                $begingroup$

                So your set is
                $$X = bigg{frac{n}{m}: n, m in mathbb{N} mbox{ and } 0 < m < nbigg}.$$
                Claim: $inf X = 0$ and $sup X = 1$. You can easily see that both of them do not sit in $X$ as $m$ cannot be $0$ and $m$ cannot be equal to $n$. So in case we prove our claim, we can say that $0$ and $1$ are not minimum and maximum of $X$. This is because maxima and minima are required to be in your set in general.
                To check that $0$ is the infimum you can check two properties: that $0$ is smaller than every element of your set, and that if you pick $varepsilon > 0$ you can always find an element of your set between $0$ and $0+varepsilon$.



                Clearly $0 < frac{m}{n}$ as $m,n > 0$. So let us find $frac{m}{n} < varepsilon$. Let us fix $l in mathbb{N}, l > 1$ large enough so that $lvarepsilon > 1$. Such an $l$ exists by the Archimedean property of the real numbers. So $frac{1}{l} < varepsilon$. This proves your first claim, $inf X = 0$.



                You can do a similar proof for $sup X = 1$. It is enough to prove that $1$ is greater than all the elements in $X$, and that once you fix $varepsilon > 0$ you can always find an element of $X$ between $1-varepsilon$ and $1$.






                share|cite|improve this answer









                $endgroup$


















                  0












                  $begingroup$

                  So your set is
                  $$X = bigg{frac{n}{m}: n, m in mathbb{N} mbox{ and } 0 < m < nbigg}.$$
                  Claim: $inf X = 0$ and $sup X = 1$. You can easily see that both of them do not sit in $X$ as $m$ cannot be $0$ and $m$ cannot be equal to $n$. So in case we prove our claim, we can say that $0$ and $1$ are not minimum and maximum of $X$. This is because maxima and minima are required to be in your set in general.
                  To check that $0$ is the infimum you can check two properties: that $0$ is smaller than every element of your set, and that if you pick $varepsilon > 0$ you can always find an element of your set between $0$ and $0+varepsilon$.



                  Clearly $0 < frac{m}{n}$ as $m,n > 0$. So let us find $frac{m}{n} < varepsilon$. Let us fix $l in mathbb{N}, l > 1$ large enough so that $lvarepsilon > 1$. Such an $l$ exists by the Archimedean property of the real numbers. So $frac{1}{l} < varepsilon$. This proves your first claim, $inf X = 0$.



                  You can do a similar proof for $sup X = 1$. It is enough to prove that $1$ is greater than all the elements in $X$, and that once you fix $varepsilon > 0$ you can always find an element of $X$ between $1-varepsilon$ and $1$.






                  share|cite|improve this answer









                  $endgroup$
















                    0












                    0








                    0





                    $begingroup$

                    So your set is
                    $$X = bigg{frac{n}{m}: n, m in mathbb{N} mbox{ and } 0 < m < nbigg}.$$
                    Claim: $inf X = 0$ and $sup X = 1$. You can easily see that both of them do not sit in $X$ as $m$ cannot be $0$ and $m$ cannot be equal to $n$. So in case we prove our claim, we can say that $0$ and $1$ are not minimum and maximum of $X$. This is because maxima and minima are required to be in your set in general.
                    To check that $0$ is the infimum you can check two properties: that $0$ is smaller than every element of your set, and that if you pick $varepsilon > 0$ you can always find an element of your set between $0$ and $0+varepsilon$.



                    Clearly $0 < frac{m}{n}$ as $m,n > 0$. So let us find $frac{m}{n} < varepsilon$. Let us fix $l in mathbb{N}, l > 1$ large enough so that $lvarepsilon > 1$. Such an $l$ exists by the Archimedean property of the real numbers. So $frac{1}{l} < varepsilon$. This proves your first claim, $inf X = 0$.



                    You can do a similar proof for $sup X = 1$. It is enough to prove that $1$ is greater than all the elements in $X$, and that once you fix $varepsilon > 0$ you can always find an element of $X$ between $1-varepsilon$ and $1$.






                    share|cite|improve this answer









                    $endgroup$



                    So your set is
                    $$X = bigg{frac{n}{m}: n, m in mathbb{N} mbox{ and } 0 < m < nbigg}.$$
                    Claim: $inf X = 0$ and $sup X = 1$. You can easily see that both of them do not sit in $X$ as $m$ cannot be $0$ and $m$ cannot be equal to $n$. So in case we prove our claim, we can say that $0$ and $1$ are not minimum and maximum of $X$. This is because maxima and minima are required to be in your set in general.
                    To check that $0$ is the infimum you can check two properties: that $0$ is smaller than every element of your set, and that if you pick $varepsilon > 0$ you can always find an element of your set between $0$ and $0+varepsilon$.



                    Clearly $0 < frac{m}{n}$ as $m,n > 0$. So let us find $frac{m}{n} < varepsilon$. Let us fix $l in mathbb{N}, l > 1$ large enough so that $lvarepsilon > 1$. Such an $l$ exists by the Archimedean property of the real numbers. So $frac{1}{l} < varepsilon$. This proves your first claim, $inf X = 0$.



                    You can do a similar proof for $sup X = 1$. It is enough to prove that $1$ is greater than all the elements in $X$, and that once you fix $varepsilon > 0$ you can always find an element of $X$ between $1-varepsilon$ and $1$.







                    share|cite|improve this answer












                    share|cite|improve this answer



                    share|cite|improve this answer










                    answered Nov 26 '17 at 17:07









                    GibbsGibbs

                    4,9123826




                    4,9123826























                        0












                        $begingroup$

                        First, we show the infimum and supernumerary are as you say. Clearly $0$ is a lower bound, and $1$ an upper bound, but we must show they are, respectively, the greatest lower bound and least upper bound. I claim that (i) if $a > 0$ is a rational number, then there exists $x in X$ such that $x < a$, and (ii) if $b < 1$ is a rational number, then there exists $y in X$ such that $y > b$.



                        For (i), let $a = m / n$, where $0 < m < n$. We wanna find a number between $0$ and $a$, so we can make the easy choice and pick the midpoint between them, namely $x = frac{m}{2n}$. Thus if $a$ was



                        For (ii), let $b = p / q, 0 < p < q$, and then let $ y = frac{2p + 1}{2 q}$. Again, we choose a point between $b$ and $1$ to show $b$ wasn't an upper bound.



                        So now we've determined that $inf X = 0, sup X = 1$. But neither $0$ nor $1$ is contained in $X$, so they can't be the minimum or maximum, respectively.






                        share|cite|improve this answer









                        $endgroup$


















                          0












                          $begingroup$

                          First, we show the infimum and supernumerary are as you say. Clearly $0$ is a lower bound, and $1$ an upper bound, but we must show they are, respectively, the greatest lower bound and least upper bound. I claim that (i) if $a > 0$ is a rational number, then there exists $x in X$ such that $x < a$, and (ii) if $b < 1$ is a rational number, then there exists $y in X$ such that $y > b$.



                          For (i), let $a = m / n$, where $0 < m < n$. We wanna find a number between $0$ and $a$, so we can make the easy choice and pick the midpoint between them, namely $x = frac{m}{2n}$. Thus if $a$ was



                          For (ii), let $b = p / q, 0 < p < q$, and then let $ y = frac{2p + 1}{2 q}$. Again, we choose a point between $b$ and $1$ to show $b$ wasn't an upper bound.



                          So now we've determined that $inf X = 0, sup X = 1$. But neither $0$ nor $1$ is contained in $X$, so they can't be the minimum or maximum, respectively.






                          share|cite|improve this answer









                          $endgroup$
















                            0












                            0








                            0





                            $begingroup$

                            First, we show the infimum and supernumerary are as you say. Clearly $0$ is a lower bound, and $1$ an upper bound, but we must show they are, respectively, the greatest lower bound and least upper bound. I claim that (i) if $a > 0$ is a rational number, then there exists $x in X$ such that $x < a$, and (ii) if $b < 1$ is a rational number, then there exists $y in X$ such that $y > b$.



                            For (i), let $a = m / n$, where $0 < m < n$. We wanna find a number between $0$ and $a$, so we can make the easy choice and pick the midpoint between them, namely $x = frac{m}{2n}$. Thus if $a$ was



                            For (ii), let $b = p / q, 0 < p < q$, and then let $ y = frac{2p + 1}{2 q}$. Again, we choose a point between $b$ and $1$ to show $b$ wasn't an upper bound.



                            So now we've determined that $inf X = 0, sup X = 1$. But neither $0$ nor $1$ is contained in $X$, so they can't be the minimum or maximum, respectively.






                            share|cite|improve this answer









                            $endgroup$



                            First, we show the infimum and supernumerary are as you say. Clearly $0$ is a lower bound, and $1$ an upper bound, but we must show they are, respectively, the greatest lower bound and least upper bound. I claim that (i) if $a > 0$ is a rational number, then there exists $x in X$ such that $x < a$, and (ii) if $b < 1$ is a rational number, then there exists $y in X$ such that $y > b$.



                            For (i), let $a = m / n$, where $0 < m < n$. We wanna find a number between $0$ and $a$, so we can make the easy choice and pick the midpoint between them, namely $x = frac{m}{2n}$. Thus if $a$ was



                            For (ii), let $b = p / q, 0 < p < q$, and then let $ y = frac{2p + 1}{2 q}$. Again, we choose a point between $b$ and $1$ to show $b$ wasn't an upper bound.



                            So now we've determined that $inf X = 0, sup X = 1$. But neither $0$ nor $1$ is contained in $X$, so they can't be the minimum or maximum, respectively.







                            share|cite|improve this answer












                            share|cite|improve this answer



                            share|cite|improve this answer










                            answered Nov 27 '17 at 3:19









                            AJYAJY

                            4,14521128




                            4,14521128






























                                draft saved

                                draft discarded




















































                                Thanks for contributing an answer to Mathematics Stack Exchange!


                                • Please be sure to answer the question. Provide details and share your research!

                                But avoid



                                • Asking for help, clarification, or responding to other answers.

                                • Making statements based on opinion; back them up with references or personal experience.


                                Use MathJax to format equations. MathJax reference.


                                To learn more, see our tips on writing great answers.




                                draft saved


                                draft discarded














                                StackExchange.ready(
                                function () {
                                StackExchange.openid.initPostLogin('.new-post-login', 'https%3a%2f%2fmath.stackexchange.com%2fquestions%2f2538207%2fhow-to-prove-supremum-and-infimum%23new-answer', 'question_page');
                                }
                                );

                                Post as a guest















                                Required, but never shown





















































                                Required, but never shown














                                Required, but never shown












                                Required, but never shown







                                Required, but never shown

































                                Required, but never shown














                                Required, but never shown












                                Required, but never shown







                                Required, but never shown







                                Popular posts from this blog

                                To store a contact into the json file from server.js file using a class in NodeJS

                                Redirect URL with Chrome Remote Debugging Android Devices

                                Dieringhausen